Search results

  • ...use it is both the median and the mode of the set. Thus <math>90</math> is correct.
    2 KB (268 words) - 18:19, 27 September 2023
  • ...idate. Multiplying <math>63\cdot64</math> confirms that our assumption is correct.
    2 KB (315 words) - 15:34, 18 June 2022
  • ...l area gives us <math>40-33\frac{1}{2}=\boxed{6\frac{1}{2}}</math>, so the correct answer is <math>\boxed{\textbf{(D)}}</math>.
    8 KB (1,016 words) - 00:17, 31 December 2023
  • ...ath>1</math> and <math>2016</math>, inclusive. Which of the following is a correct statement about the probability <math>p</math> that the product of the thre
    2 KB (297 words) - 14:54, 25 June 2023
  • ...ectly answer the question receives one point. The person who gets the most correct out of three questions (not necessarily two out of three) is the winner. ...ritten competitors. Starting then, the first person to get three questions correct wins (as opposed to the best-out-of-three rule).
    10 KB (1,497 words) - 11:42, 10 March 2024
  • ...econd minute 4 points, or third minute 3 points. The first team to get the correct answer receives a bonus of 2 points. ...rs are worth 2 points each. Bonus points are given if all four answers are correct and the answer sheet is submitted at the 3 minute mark (First Command – 5
    8 KB (1,182 words) - 14:26, 3 April 2024
  • ...ation competition, a mental math competition (where only exact answers are correct), and a physical estimation competition. Grades 4-9 compete in this format.
    2 KB (216 words) - 16:59, 17 December 2016
  • A correct solution would be to move everything to the left side of the inequality, an
    12 KB (1,798 words) - 16:20, 14 March 2023
  • ...blems are graded on a scale from 0 to 5. The full 5 points are awarded for correct, concise solutions, and points are taken off from there for mistakes or oth
    4 KB (613 words) - 13:08, 18 July 2023
  • The AMC 10 is scored in a way that penalizes guesses. Correct answers are worth 6 points, incorrect questions are worth 0 points, and una
    4 KB (574 words) - 15:28, 22 February 2024
  • The AMC 12 is scored in a way that penalizes guessing. Correct answers are worth 6 points, incorrect answers are worth 0 points, and unans
    4 KB (520 words) - 12:11, 13 March 2024
  • ...ve, making guessing almost futile. Wrong answers receive no credit, while correct answers receive one point of credit, making the maximum score 15. Problems ...inally, additions to and improvements on the solutions in the AoPSWiki are always welcome.
    8 KB (1,057 words) - 12:02, 25 February 2024
  • *4 points for each correct question *5 points for each correct question
    4 KB (632 words) - 17:09, 11 October 2020
  • The test is scored as 5 points for every correct response, 1 point for a blank response, and 0 points for an incorrect respo
    972 bytes (141 words) - 11:12, 30 September 2018
  • ...pping the problem can help you check and solve other problems, and you can always return to the problem that you skipped. This concept helps double where yo ...good guess, saving you time and giving you a chance of getting the answer correct. This tactic helps the most in competitions with multiple choice answers. I
    3 KB (538 words) - 13:13, 16 January 2021
  • ...e triangle. It is easy to verify that this placement of the orthocenter is correct and that the orthic triangle will remain the same as before the swapping, a
    8 KB (1,408 words) - 11:54, 8 December 2021
  • ...rcount several numbers, such as 12, which is divisible by both 2 and 3. To correct for this overcounting, we must subtract out the numbers that are divisible Another basic example is combinations. In these, we correct for overcounting with division, by dividing out what we overcount (as oppos
    4 KB (635 words) - 12:19, 2 January 2022
  • * 4 points for each correct answer ...consists of 5 short-answer questions. Your score is 5 times the number of correct answers, for a maximum score of 25.
    4 KB (644 words) - 12:56, 29 March 2017
  • Each correct answer will earn 5 points, a question left blank will earn 1 point, and an
    2 KB (366 words) - 22:59, 17 November 2007
  • ...> [[factor]]s, two of them are small. If we want to make sure that this is correct, we could test with a smaller number, like <math>30</math>. It becomes much
    2 KB (276 words) - 05:25, 9 December 2023
  • ...le (inclusive and integers only). Full credit is only given for complete, correct solutions. Each solution is intended to be in the form of a [[proof writin
    3 KB (490 words) - 03:32, 23 July 2023
  • For every correct answer: 5 points
    3 KB (388 words) - 23:07, 5 February 2024
  • ...inally, additions to and improvements on the solutions in the AoPSWiki are always welcome.
    3 KB (391 words) - 16:00, 21 February 2024
  • On the AMC 12, each correct answer is worth <math>6</math> points, each incorrect answer is worth <math
    13 KB (1,953 words) - 00:31, 26 January 2023
  • ...ssumes he is the first to arrive. If each takes what he believes to be the correct share of candy, what fraction of the candy goes unclaimed?
    13 KB (1,955 words) - 21:06, 19 August 2023
  • ..., and <math>R</math> the area of the red square. Which of the following is correct?
    12 KB (1,792 words) - 13:06, 19 February 2020
  • Cassandra sets her watch to the correct time at noon. At the actual time of 1:00 PM, she notices that her watch rea
    13 KB (1,987 words) - 18:53, 10 December 2022
  • Inductive Step: Suppose the formula is correct for <math>z_k</math>, then ...z|</math>. Therefore, the magnitude of <math>\frac{iz_n}{|z_n|}</math> is always <math>1</math>, meaning that all of the numbers in the sequence <math>z_k</
    4 KB (660 words) - 17:40, 24 January 2021
  • ...4}</math> But, this over-counts since it counts numbers like 0213. We can correct for this over-counting. Lock the first digit as 0 and permute 3 other chose
    3 KB (562 words) - 18:12, 4 March 2022
  • ...clear from the problem setup that <math>0<\theta<\frac\pi2</math>, so the correct value is <math>\tan(\theta)=\frac53</math>. Next, extend rays <math>\overri
    9 KB (1,501 words) - 05:34, 30 October 2023
  • ...the formula <math>s=30+4c-w</math>, where <math>c</math> is the number of correct answers and <math>w</math> is the number of wrong answers. Students are not
    6 KB (933 words) - 01:15, 19 June 2022
  • ...ailable ten-button lock may be opened by depressing -- in any order -- the correct five buttons. The sample shown below has <math>\{1, 2, 3, 6, 9\}</math> as
    6 KB (902 words) - 08:57, 19 June 2021
  • ...bsent-minded professor failed to notice that his calculator was not in the correct angular mode. He was lucky to get the right answer. The two least positive
    7 KB (1,177 words) - 15:42, 11 August 2023
  • ...the formula <math>s=30+4c-w</math>, where <math>c</math> is the number of correct answers and <math>w</math> is the number of wrong answers. (Students are no Let Mary's score, number correct, and number wrong be <math>s,c,w</math> respectively. Then
    7 KB (1,163 words) - 23:53, 28 March 2022
  • Since this is an AIME problem, there is exactly one correct answer, and thus, exactly one possible value of <math>f(14,52)</math>.
    4 KB (538 words) - 13:24, 12 October 2021
  • ...available ten-button lock may be opened by pressing -- in any order -- the correct five buttons. The sample shown below has <math>\{1,2,3,6,9\}</math> as its
    1 KB (181 words) - 18:23, 26 August 2019
  • ...As no other option choice fits, <math>\boxed{\textbf{(A)}-x}</math> is the correct solution.
    1 KB (179 words) - 10:33, 19 August 2022
  • THIS SOLUTION IS INCORRECT, PLEASE CORRECT IT IF YOU HAVE TIME!
    3 KB (447 words) - 17:02, 24 November 2023
  • ...r of <math>11</math> is <math>(5,11)</math>, and checking shows that it is correct.
    4 KB (628 words) - 22:05, 7 June 2021
  • ...way to do the problem is by the process of elimination. The only possible correct choices are the highest powers of each prime, <math>2^3=8</math>, <math>3^2
    5 KB (878 words) - 14:39, 3 December 2023
  • ...th> to <math>96</math> alternate in fake-real-fake-real, where we have the correct order of cards once the first <math>96</math> have moved and we can start p ...the spacing of the cards moved, <math>a</math> is an integer such that the correct first card is moved, and <math>k</math> is an integer greater than or equal
    15 KB (2,673 words) - 19:16, 6 January 2024
  • ...ing <math>2</math> and <math>7</math> for <math>w_1</math> does not give a correct product. Thus, <math>\frac{27}{50}</math> must be a reduced form of the ac
    7 KB (1,011 words) - 20:09, 4 January 2024
  • ...ination counts only one of the permutations; we can say that it counts the correct (ascending order) permutation. ...sformations of the problem, a recursion formula can be a robust way to the correct answer.
    11 KB (1,729 words) - 20:50, 28 November 2023
  • ...th> and <math>C</math>. Oh wait they are symmetric. So then if this is the correct answer, why am I wrong, or what happened to that factor of <math>3</math>?"
    15 KB (2,406 words) - 23:56, 23 November 2023
  • ...bsent-minded professor failed to notice that his calculator was not in the correct angular mode. He was lucky to get the right answer. The two least positive
    2 KB (336 words) - 19:30, 24 June 2020
  • ...math>11</math> while <math>(a-b)</math> is a factor of nine (1 or 9). The correct guesses are <math>a = 6, b = 5</math> causing <math>x = 65, y = 56,</math>
    5 KB (845 words) - 19:23, 17 September 2023
  • ...h>, we see <math>6\cdot8=30+18</math>. Therefore, we can see our answer is correct.
    1 KB (155 words) - 17:30, 16 December 2021
  • Thus, the correct answer is <math>\boxed{\textbf{(C) }3}.</math> Note to readers: make sure to always read the problem VERY carefully before attempting; it could mean the differ
    3 KB (450 words) - 02:00, 13 January 2024
  • ...= 72</math>. 28 is not divisible by 3, so we know that this number is not correct. Moving on to 7, <math>13 \cdot 7 = 91</math>. We know that 9 is a multiple
    3 KB (429 words) - 18:14, 26 September 2020
  • The test is scored as 1 point for each correct answer, 0 points for blank answers, and -.25 for incorrect answers except f
    2 KB (365 words) - 21:21, 18 March 2017
  • ...lifying of the brackets. Open the brackets and you should notice why it is correct. If you are also wondering whether or not if we got all the sets in the abo
    2 KB (263 words) - 18:13, 19 October 2021
  • ...rn techniques. However, a flaw was discovered soon after. Wiles managed to correct the proof by October 1994, thus solving the last of Fermat's problems. It i
    5 KB (860 words) - 17:10, 21 March 2023
  • The [[Clay Mathematics Institute]] has offered a USD \$1,000,000 prize for a correct solution, as it has listed it as one of its [[Millennium Problems]].
    6 KB (1,104 words) - 15:11, 25 October 2017
  • ...given side of <math>A'</math> on the straight line <math>a'</math>, we can always find one and only one point <math>B'</math> so that the segment <math>AB</m Every segment is congruent to itself; that is, we always have
    10 KB (1,655 words) - 21:43, 24 March 2022
  • So our correct answer choice is <math>\boxed{\textbf{(B) }\frac{2}{5}x^2}</math>
    2 KB (265 words) - 19:07, 25 December 2022
  • ...c{1}{1} + \frac{1}{2} - \frac{1}{6}\right) = 2</math>, which we know to be correct.
    3 KB (473 words) - 12:57, 20 February 2024
  • ...ive minute, twenty-five question multiple-choice test worth six points per correct response, and ten ciphering questions. Each ciphering question is worth te
    2 KB (307 words) - 20:25, 12 March 2012
  • ...s. Every test case has equal weight within the problem. For each test case correct, you receive the points for the test case. Programs get 4 seconds per test Each test case will give feedback on how well your program did. Correct test cases show how much time and memory used. Incorrect test cases are dif
    2 KB (297 words) - 01:41, 21 January 2023
  • ...ts must answer 30 multiple choice mathematics problems in 60 minutes. Each correct answer gives 4 points, each wrong answer subtracts 1 point and no answer gi
    2 KB (366 words) - 14:26, 4 September 2017
  • Altogether, Tori answered <math>7 + 12 + 21 = 40</math> questions correct. ...e needed to answer <math>45 - 40 = 5</math> more questions to pass, so the correct answer is <math>\boxed{(B) 5}</math>
    1 KB (167 words) - 20:30, 11 January 2024
  • ...ciple of Relativity says it is impossible to ascertain that one of them is correct while the other is wrong.
    3 KB (538 words) - 23:39, 2 March 2008
  • On a twenty-question test, each correct answer is worth 5 points, each unanswered question is worth 1 point and eac
    13 KB (1,994 words) - 13:04, 18 February 2024
  • ...ng questions, each of which contestants have three minutes to answer. Each correct answer is worth five points. Each team's score is determined by the top fou
    3 KB (475 words) - 21:51, 31 December 2013
  • ...twenty-five question multiple-choice test. Four points are given for each correct answer, and 1.1 points are given for each blank answer. On this round a [[
    2 KB (324 words) - 10:23, 24 April 2007
  • ...ears. The tests consist of 25 questions which are worth 4 point each for a correct answer, minus 1 point for an incorrect answer. Ciphering rounds- 4 of them,
    1 KB (160 words) - 12:31, 27 December 2006
  • ...ons and five free-response questions; a student's score is five points per correct questions, plus one point per answer left blank; there is no penalty for in
    2 KB (353 words) - 17:02, 7 June 2009
  • ...nior high division; a student's score is forty points plus four points per correct question minus one point per incorrect question. No calculators are permit
    1 KB (154 words) - 09:45, 21 March 2010
  • ...o we divide the whole sum by 3 and we add or subtract <math>q(n)</math> to correct for the integer based on the modularity of the sum with 3
    4 KB (595 words) - 12:14, 25 November 2023
  • The answer is clearly correct, but the proof has a gap, i.e. there is no reason that <math>f(-2)\neq1</ma
    7 KB (1,335 words) - 17:44, 25 January 2022
  • All possibilities yield a contradiction, so our assumption can not be correct.
    3 KB (438 words) - 01:19, 27 December 2023
  • ...ac{1}{\sqrt{2006}}-\frac{1}{\sqrt{2007}}</math>, which of the following is correct?
    13 KB (1,990 words) - 08:29, 19 December 2009
  • ...ac{1}{\sqrt{2006}}-\frac{1}{\sqrt{2007}}</math>, which of the following is correct?
    808 bytes (139 words) - 16:56, 6 May 2007
  • Which of the following is correct, about the graph of <math>f</math>? ...and <math>y=\sqrt[3]{6}-\sqrt[3]{3}</math>, then which of the following is correct?
    11 KB (1,672 words) - 10:56, 27 April 2008
  • ...s have three problems to solve in five hours. Students with three to four correct problems are usually invited to the final round, which has the same format
    1 KB (223 words) - 22:34, 2 January 2008
  • The 2007 AMC 12 contests will be scored by awarding 6 points for each correct response, 0 points for each incorrect response, and 1.5 points for each pro
    864 bytes (121 words) - 10:50, 4 July 2013
  • Which of the following is correct, about the graph of <math>f</math>? From above, <math>\mathrm{(A)}</math> is not correct because the graph does not intersect the x-axis (it is tangent to it).
    1 KB (216 words) - 10:46, 27 April 2008
  • ...and <math>y=\sqrt[3]{6}-\sqrt[3]{3}</math>, then which of the following is correct?
    1 KB (182 words) - 10:40, 27 April 2008
  • ...ind out that out of the 9 cases, in 4 the value <math>n_{?}</math> has the correct sum of digits. <br/> ...s of whether the sum carries or not, the modulo 9 of the sum of the digits always increases by <math>1</math>.
    15 KB (2,558 words) - 19:33, 4 February 2024
  • ...inally, additions to and improvements on the solutions in the AoPSWiki are always welcome.
    924 bytes (121 words) - 00:41, 12 October 2013
  • The 2007 AMC 12 contests will be scored by awarding 6 points for each correct response, 0 points for each incorrect response, and 1.5 points for each pro
    12 KB (1,814 words) - 12:58, 19 February 2020
  • ...; the number that Cindy started with is <math>3(43)+9=138</math>. Now, the correct result is <math>\frac{138-3}{9}=\frac{135}{9}=15</math>. Our answer is <mat ...ath>. Solve for <math>x</math> gives us <math>x=138</math>. Therefore, the correct result is <math>\frac{138-3}{9}=\frac{135}{9}=\boxed{\textbf{(A) }15}</math
    1 KB (163 words) - 12:46, 8 November 2021
  • Which one of the following must necessarily be correct?
    13 KB (1,945 words) - 18:28, 19 June 2023
  • On the AMC 12, each correct answer is worth <math>6</math> points, each incorrect answer is worth <math
    837 bytes (122 words) - 20:14, 3 July 2013
  • ...-[[density]], low-pressure gases. For higher densities, it is necessary to correct this equation. For greater precision, the [[van der Waals equation]] is ano
    1 KB (225 words) - 08:59, 11 March 2008
  • ...whence we immediately obtain <math>\framebox[1.2\width]{(A)}</math> as the correct answer.
    5 KB (814 words) - 18:02, 17 January 2023
  • Therefore the correct answer is <math>\mathrm{(E)}</math>
    2 KB (268 words) - 12:40, 3 June 2021
  • The addition below is incorrect. The display can be made correct by changing one digit <math>d</math>, wherever it occurs, to another digit
    17 KB (2,387 words) - 22:44, 26 May 2021
  • <math> \textbf{(B)}\ \text{In some cases there is more than one correct order in proving certain propositions.} </math> <math> \textbf{(D)}\ \text{It is not possible to arrive by correct reasoning at a true conclusion if, in the given, there is an untrue proposi
    23 KB (3,641 words) - 22:23, 3 November 2023
  • For all integers x, <math>x^2</math> is always a positive integer. So solve for <math>\frac{n}{20-n} = 0</math>, getting < ...hat also yield an integer n value, meaning that there are 4 values, so the correct answer is <math>\boxed{(D)}</math>
    4 KB (579 words) - 05:54, 17 October 2023
  • ...e of <math>2004</math>, so there is a very high probability that it is the correct answer.
    3 KB (533 words) - 14:52, 29 October 2023
  • Subtracting that from 1 to get the probability she can park, the correct answer is <math>\boxed{E}</math>.
    4 KB (653 words) - 11:06, 15 October 2022
  • ...Indeed, note that <math>a(1,k) = 2^{1-1}(1+2k-2)=2k-1</math>, which is the correct formula for the first row. We claim the result by [[induction]] on <math>n<
    3 KB (509 words) - 17:21, 22 March 2018
  • ...inally, additions to and improvements on the solutions in the AoPSWiki are always welcome.
    1 KB (115 words) - 13:51, 1 May 2008
  • |4 points per correct answer, 0 points per blank answer, -1 points per incorrect answer |1 point per correct answer, 0 points otherwise
    5 KB (801 words) - 12:47, 23 September 2023
  • ...<math>a+b+c+d=7+5+11+8=31</math>. <math>\boxed{\textbf{(B)}\ 31}</math> is correct.
    6 KB (914 words) - 11:07, 7 September 2023
  • The addition below is incorrect. The display can be made correct by changing one digit <math>d</math>, wherever it occurs, to another digit ...either a <math>5</math> (no carry) or a <math>6</math> (carry) to create a correct statement.
    1 KB (223 words) - 13:59, 5 July 2013
  • ...of them as soon as the mathematical community had pronounced his solution correct. All but one of these problems had been solved by the meeting in 2000, and
    13 KB (1,969 words) - 17:57, 22 February 2024
  • Michael plays catcher for his school's baseball team. He has always been a great player behind the plate, but this year as a junior, Michael's Joshua finds an answer which Michael confirms is correct. What is Joshua's correct answer (the units digit of <math>2008^{2008}</math>)?
    71 KB (11,749 words) - 01:31, 2 November 2023
  • ...e are what you really need to compile it - the first 2 are to get into the correct folder), in the terminal, run:
    5 KB (732 words) - 00:47, 13 December 2023
  • have divided the number by 2 to get the correct answer. What is the correct
    13 KB (1,821 words) - 22:18, 5 December 2023

View (previous 100 | next 100) (20 | 50 | 100 | 250 | 500)